Asia is the largest of the world’s seven continents with an area of approximately 1.7 times 10 Superscript 7 square miles. Australia, the smallest continent, has an area of approximately 3.1 times 10 Superscript 6 square miles.

How much larger is Asia than Australia? Express the answer in scientific notation.
1.40 times 10 Superscript 6 square miles
2.93 times 10 Superscript 6 square miles
1.39 times 10 Superscript 7 square miles
2.01 times 10 Superscript 7 square miles

Asia Is The Largest Of The Worlds Seven Continents With An Area Of Approximately 1.7 Times 10 Superscript

Answers

Answer 1

The amount of area larger is Asia than Australia will be 1.39 × 10⁷. Then the correct option is C.

What is subtraction?

It simply implies subtracting something from an entity, group, location, etc. Subtracting from a collection or a list of ways is known as subtraction.

Asia is the largest of the world’s seven continents, with an area of approximately 1.7 times 10⁷ square miles.

Australia, the smallest continent, has an area of approximately 3.1 times 10⁶ square miles.

Then the amount of area larger is Asia than Australia will be

⇒ 1.7 × 10⁷ – 3.1 × 10⁶

⇒ 1.7 × 10⁷ – 0.31 × 10⁷

⇒ 1.39 × 10⁷

Then the correct option is C.

More about the subtraction link is given below.

https://brainly.com/question/4319655

#SPJ1


Related Questions

700=132.69x-25.96
solve please

Answers

Answer:

x = 5.5 (rounded)

Step-by-step explanation:

Equation: 700 = 132.69x - 25.96

Add 25.96 to both sides: 700+25.96 = 132.69x -25.96 + 25.96

Simplify: 725.96 = 132.69x

Isolate x

Divided both sides by 132.69: [tex]\frac{725.96}{132.69} = \frac{132.69x}{132.69}[/tex]

Simplify: x = 5.5 (rounded)

Julie is solving the equation x2 + 5x+6= 0 and notices that the discriminant b2- 4ac has a value of 1. This tells her that the equation
has
A) no real roots

B) exactly one real root.

C) exactly two real roots.

D) exactly three real roots.

Answers

Answer: C

Step-by-step explanation:

If you use the quadratic equation you will find that there are two possible answers to this problem

The explanation shows that the equation has exactly two real roots. so the correct option is C.

What is a quadratic equation?

A quadratic equation is the second-order degree algebraic expression in a variable. the standard form of this expression is  ax² + bx + c = 0 where a. b are coefficients and x is the variable and c is a constant.

Given that Julie is solving the equation x² + 5x+6= 0 and notices that the discriminant b²- 4ac has a value of 1.

x² + 5x+6= 0

The discriminant = b²- 4ac = 1

It will be Real and distinct because the discriminant is positive.

There are two roots of a quadratic equations always if  the discriminant is positive. We say that the quadratic equation has 2 solution if roots are distinct, and have 1 solutions when both roots are same.

Hence, This tells her that the equation has exactly two real roots. so the correct option is C.

Learn more about quadratic equations;

https://brainly.com/question/17177510

#SPJ2

In circle F with mEHG = 42, find the mEFG.

Answers

Answer:

∠EFG=2×∠EHG

∠EFG=2×42

∠EFG=84°

so,m∠EFG=84°

------------------------

hope it helps...

have a nice day!!

Gerry is painting a fence. He painted
of the fence in the morning and of
the fence in the afternoon. How much
of the fence has he painted in all?
Give your answer in simplest form.
of the fence
Enter Plz help

Answers

17\20 should be the correct answer

PLEASE help!! I will give BRAINLIST :)

Answers

Answer:

[tex]1 + cot^2 \theta[/tex]

Step-by-step explanation:

Given the expression;

[tex]\frac{tan^2\theta+1}{tan^2 \theta}[/tex]

Separating into partial fraction;

[tex]\frac{tan^2 \theta}{tan^2 \theta} + \frac{1}{tan^2 \theta}\\= 1 + \frac{1}{tan^2 \theta}\\\\[/tex]

Since [tex]\frac{1}{tan \theta} = cot \theta\\[/tex]

Hence the expression becomes;

[tex]1 + cot^2 \theta[/tex]

11. Square ABCD shown has sides of length 10 centimeters. The unshaded
portions are both semicircles. Calculate the area of the shaded portion to
the nearest tenth of a square centimeter. *

Answers

Answer:

21cm^2

Step-by-step explanation:

Area of Square-Area of semi circles (makes one full circle)

Squares Area = 100

LxW or (10x10)=100

Circle Area = 25/pi

/pi (radius)^2 or /pi(5)^2 = 25/pi

100cm^2-25cm^2 = 21.4601…cm^2 rounded to 21.5 cm^2

The area of the shaded region is 21.5 cm²

What is a semi-circle?

'In geometry, a semicircle is a plane figure that is formed by dividing a circle into exactly two parts.'

According to the given problem,

Side length of the square = 10

Area of the square = ( 10 × 10 )

                               = 100 cm²

Now, for the semi-circle,

Diameter = 10 cm

Radius = [tex](\frac{10}{2}[/tex][tex])[/tex] cm

            = 5 cm

Area of the semicircle =  [tex]\frac{\pi r^{2} }{2}[/tex]

                                     = [tex]\frac{\pi * 5^{2} }{2}[/tex]

Since, there are two semicircles in the square, we multiply the area with 2,

⇒ [tex]\frac{\pi *5^{2} }{2}*2[/tex]

= 25[tex]\pi[/tex]

Area of the shaded region = ( 100 - 25π )

                                            = ( 100 - 78.53 )

                                            = 21.46

                                            ≈ 21.5 cm²

Hence, we can conclude, the area of the shaded region is 21.5 cm².

Learn more about semi-circles here: https://brainly.com/question/14429410

#SPJ2

Can anyone help me with this please? I need help along with an explanation.

Answers

the value of a/b = 2.5 because a = 5 and b = 2.

if a^2 - b^2 = 21,
5^2 = 25, 2^2 = 4,
25 - 4 = 21

if a - b = 3,
5 - 2 = 3

5/2 = 2.5

hope this helps! :)

Answer:

a + b = 7

Step-by-step explanation:

a^2 - b^2 = 21

Factor the left side. It is the difference of two squares which factors into the product of a sum and a difference.

(a + b)(a - b) = 21

We are told a - b = 3, so substitute 3 for the factor a - b.

(a + b)(3) = 21

Divide both sides by 3.

a + b = 7


[tex] {( \frac{5}{3} )}^{2n + 1} {( \frac{5}{3} )}^{5} = ({ \frac{5}{3} )}^{n + 2} [/tex]
Pls include steps...

Answers

Answer:

n = -4

Step-by-step explanation:

[tex](\frac{5}{3} )^2^n^+^1 * (\frac{5}{3} )^5 = (\frac{5}{3})^n^+^2[/tex]

in multiplication if the base are same u can add there exponent.

[tex](\frac{5}{3})^2^n^+^1^+^5 = (\frac{5}{3})^n^+^2[/tex]

base of both sides are equal so their exponent will be equal

2n + 1 + 5 = n + 2

2n + 6 = n + 2

2n - n = 2 - 6

n = -4

Middle School teaches 6th, 7th, and 8th grade classes. The sixth grade has 318 students, the 7th has 286 students, and the 8th has 306 students. If the sixth grade takes a field trip and each student needs $3.50, how much total money is needed by the students? $910 c. $1113 b. $3185 d. $1071​

Answers

Answer:

$3185

Step-by-step explanation:

Multiply the number of grade 6 students by the money needed ($3.50) and you get 318x3.50=1113

Multiply the number of grade 7 students by the money needed ($3.50) and you get  286x3.50=1001

Multiply the number of grade 8 students by the money needed ($3.50) and you get 306x3.50=1071

Add all the total amounts together and you get $3185

A company paid a ₦25200 electricity bill to NEPA. The bill included VAT at 5%. Calculate the amount of VAT paid​

Answers

Answer:

1260

Step-by-step explanation:

5÷100=0.05

0.05×25200=1260

Which quotient is greater than 3? 28 divided by 10, 35 divided by 11, 38 divided by 13 or 40 divided by 14

Answers

Simply divide these all! 28/10 is 2.8, 35/11 is 3.1, 38/13 is 2.9, and 40/14 is 2.8! 35/11 is the only one that is more than 3

The quotient 35 divided by 11 is the only one among the options that is greater than 3.

To determine which quotient is greater than 3, we can calculate the value of each quotient:

28 divided by 10 = 2.8

35 divided by 11 ≈ 3.182

38 divided by 13 ≈ 2.923

40 divided by 14 ≈ 2.857

Comparing these values, we can see that only the quotient 35 divided by 11, which is approximately 3.182, is greater than 3.

Therefore, the quotient 35 divided by 11 is the only one among the options that is greater than 3.

To know more about divided:

https://brainly.com/question/15381501


#SPJ6

Jamil’s car uses 4 and 2/3
gallons of gasoline to travel 49 miles. At this rate how far can Jamil’s car travel per gallon of gasoline?

Answers

I’m also stuck on this question

Answer:

10.5 miles

Step-by-step explanation:

[tex]\frac{4\frac{2}{3} }{49} :\frac{1}{y}[/tex]

4 2/3 × y = 1 × 49

[tex]4\frac{2}{3}y=49[/tex]

[tex]4\frac{2}{3}y/4\frac{2}{3} =49/4\frac{2}{3}[/tex]

[tex]y = 10\frac{1}{2}[/tex]

10/d(d-2) + 4/d = 5/d-2

Answers

Answer:

impossible

Step-by-step explanation:

a denominator can’t be equal to 0. So:

d ≠ 0 ; d ≠ 2

(10 + 4d - 8)/d(d-2) = 5d/ d(d-2)

4d + 2 = 5d

d = 2

But we have said that d can’t be equal to 2, so the equation is impossible.

what is the domain of the function g(x)
pls help what are the answers a through f

Answers

Answer:

Domain : (-∞, ∞)

Step-by-step explanation:

Domain of a function is a set of x-values of the function.

Therefore, all x-values at which the given function is defined will be the domain of the function.

From the graph attached,

Given function is defined for all x-values.

Domain : Set of all real numbers.

Or Domain : (-∞, ∞)

can someone please answer

Answers

Answer:

x =62

Step-by-step explanation:

The two angles form a straight line so they add to 180

x+118 = 180

Subtract 118 from each side

x = 180-118

x =62

Answer:

x = 62°

Step-by-step explanation:

These both angles are located at straight line.

So, the sum of both angles is 180 °.

x + 118 ° = 180 °

Subtract 118 ° from 180 °

x = 180 ° - 118 °

x = 62 °

Find the sale price of an $18 item after a 50% discount.

Answers

Rewrite 50% as a decimal : 0.50

Multiply the price by 0.50:

18 x 0.50 = 9

The sale price is $9

Answer:

$9

Step-by-step explanation:

price of an item=$18

discount=50%

sale price =? (be x)

sale price= original price -discount% of original price

x=$18 -50/100 * $18

x=$1800-$900/100

=$900/100

=$9

therefore sale price of an item is $9.

Berto has $12 to put gas in his car. If gas costs $3.75 per gallon, which ordered pair relating number of gallons of gas, x, to the total cost of the gas, y, includes the greatest amount of gas Berto can buy?



(__,__ )

Answers

Answer:

If gas costs $3.75 per gallon and Berto has $12, then he can purchase 12/3.75 gallons. This is approximately 3.2 gallons. So the coordinate on this line would be (3.2, 12).

Actually... It's from web

the answer is (3.2,12)

If j=h and k=m then which expression represents the value of g

Answers

Answer:

Step-by-step explanation:

as given j=h and k=m then which expression for g is

as from attached image , we can see that its look like two right angle triangle

so from lower triangle we apply the pythagorean theorem .

which state that sum of the squares on the legs of a right triangle is equal to the square on the hypotenuse.

so small triangle ΔEBD

h² +k²= g²       ---------------(1)

now from big triangle ΔABC

(j + h)² +(m + h)² =f²---------(2)

given j=h and k=m so from equation-2 we can find ,

(h + h)² +(k+ k)² =f²

(2h)² +(2k)²=f²

4h²+ 4k²= f²

4(h²+k²)= f² ----------(3)

now from equation 1 and 3 we can obtained .

4×g² =f²

take square root both side we get,

2×g=f

so g=f/2----- Answer

what are ascending number's​

Answers

Answer:

Ascending order

Step-by-step explanation:

Ascending numbers are numbers that begin from smallest to biggest.

What is 300+3x40-25x4=?

Answers

Answer:

320

Step-by-step explanation:

300+3x40=120-25x4

300+120-25x4=100

320

Answer:

300+3x40-25x4 = 320

Step-by-step explanation:

300+3x40-25x4

= 300+ 120 - 100

= 420-100

= 320

help me please this is due at 10

Answers

Answer:

3,375mm^3

Step-by-step explanation:

To find the volume, you do the length times the width times the height. Since this is a cube, the length, width, and height are all 15 mm. Multiply 15 times 15 times 15 to get 3375. The answer is 3375 cubic inches or 3375mm^3

SUPER URGENT: What is the range of y = sinx in the interval - π ≤ x ≤ 0?
-1 ≤ y ≤ 1
-1 ≤ y ≤ 0
0 ≤ y ≤ 1
y ≤ 1

Answers

Answer:

-1 ≤ y ≤ 0

Step-by-step explanation:

here, we simply want to know the range of values for sin x over the given interval

The range of values for sin x are simply the values in which y will take over the given interval of x

let’s start with the value sine (-pi)

We have this as;

0

And the value of sin (0) = 0

Recall, between -pi and 0, we have (-pi/2)

so sin (-pi/2) = -1

Thus, we have the correct range of values as;

-1 ≤ y ≤ 0

Why is it easy to hammer a sharp pin than a blunt pin ? *

Answers

Answer:

It is easier to hammer a sharp nail than a blunt one

Step-by-step explanation:

This is because blunt nail occupies more area than sharped nail. As sharp will exert more pressure as it occupies less area.

please help soon, i can't seem to figure this one out!!

Consider the following equation. -2x + 6 = -(2/3)^x + 5. Approximate the solution to the equation above using three iterations of successive approximation. Use the graph below as a starting point.

A. x = 3/4
B. x = 13/16
C. x = 7/8
D. x = 15/16

Answers

I think the answer is B

The required solution after three successive iterations is near x = 7/8. Option c is correct.

Equation is -2x + 6 = -(2/3)^x + 5. Approximate the solution to the equation above using three iterations of successive approximation.

What is the equation?

The equation is the values of two expressions that are equal.

Here,

[tex]-2x + 6 = -(2/3)^x + 5[/tex]

Arranging the equation

[tex]-2x + 6 +(2/3)^x - 5 = 0\\(2/3)^x-2x+1=0[/tex]
Since the solution of the equation is when f(x) = 0 at near to  x = 0.8
[tex]f(x) = (2/3)^x-2x+1\\[/tex]

First iteration at x =0.8
[tex]f(0.8) = (2/3)^x-2x+1\\ = (2/3)^{0.8}-2*0.8+1\\ = 0.123[/tex]

It will go up more to get exact zero, so
Second iteration at x = 0.9

[tex]f(0.9) = (2/3)^{0.9}-2*0.9+1\\f(0.9) = -0.106[/tex]

It seems that zero is near 0.9 so will go down
The third iteration at x = 0.87

[tex]f(0.87) = (2/3)^{0.87}-2*0.87+1\\f(0.87) = -0.037[/tex]

Here , solution is much near to x = 0.87 or x =7/8.


Thus, the required solution after three successive iterations is near x = 7/8.

Learn more about the equation here:
https://brainly.com/question/10413253

#SPJ5

can someone help i dont understand exponential functions

Answers

Answer:

1,3,4,6

Step-by-step explanation:

solve for x
4^5x=(1/32)^1-x

Answers

x = -1

Step-by-step explanation:

[tex] {4}^{5x} = {2}^{2(5x)} = {2}^{10x} [/tex]

[tex] {( \frac{1}{32}) }^{(1 - x)} = {2}^{ - 5(1 - x)} = {2}^{(5x - 5)} [/tex]

or

[tex] {2}^{10x} = {2}^{(5x - 5)} [/tex]

Since both sides have the same base, we can write

10x = 5x - 5

or

5x = -5

x = -1

Can someone please help me create an equation for this circle?

Answers

Answer:

[tex](x-1)^2 + (y-1)^2 = 10[/tex]

Step-by-step explanation:

From the graph ,

Centre of the circle is (1, 1)

To find radius we find a point through which the circle passes and find the distance between the Centre and the point.

Let that point be (4, 0)

[tex]radius = \sqrt{(x_2 - x_1)^2 + (y_2-y_1)^2} = \sqrt{(1-4)^2 + (1-0)^2} = \sqrt{10}[/tex]

Equation of the circle :

[tex](x - a)^2 + (y-b)^2 = r^2 \ , \ where \ (a,b)\ is\ the\ centre\ of\ the\ circle\ and\ r\ is\ the\ radius\\\\(x-1)^2 + (y-1)^2 = 10[/tex]

ok pls pls help !!!!!!!!!!!!!!!!!!!!!!!!

Answers

Step-by-step explanation:

everything can be found in the picture

A music store bought a CD set at a cost of​ $20. When the store sold the CD​ set, the percent markup was​ 40%. Find the selling price.

Answers

Answer:

$28.00

Step-by-step explanation:

1. The markup is 40% of the $20 cost, so the markup is:

(0.40)(20) = 8

2. Then the selling price, being the cost plus markup, is:

20 + 8 = 28

3. The CD set sold for $28.00

Step-by-step explanation:

sp=(100+p)% of cp

=100+40/100*20

=140/100*20

=28

I'LL MARK BRAINLIEST !!!!


Select the two correct solutions of the inequality x + 8 > 14.


a. 2


b. 4


c. 6


d. 8


e. 10


PLEASE ANSWER QUICKLY !!

Answers

Answer:

d and e

Step-by-step explanation:

for a

substitute 2 for x

2 + 8 > 14

2 + 8 = 10

10 > 14

10 is not greater than 14 therefore 2 is not a solution to the inequality

for b

substitute 4 for x

4 + 8 > 14

4 + 8 = 12

12 > 14

12 is not greater than 14 therefore 4 is not a solution to the inequality

for c

substitute 6 for x

6 + 8 > 14

6 + 8 = 14

14 > 14

14 is not greater than 14 therefore 6 is not a solution to the inequality

for d

substitute 8 for x

8 + 8 > 14

8 + 8 = 16

16 > 14

16 is greater than 14 therefore 8 is a solution to the inequality

for e

substitute 10 for x

10 + 8 > 14

10 + 8 = 18

18 > 14

18 is greater than 14 therefore 10 is a solution to the inequality

we can conclude that the answers are d and e

Other Questions
Please help? I really need it Find the value of x. Returning from Vietnam, we were indeed given a parade.Crowds of screaming people waving signs - not just onone road, one day. No, they were everywhere. Every day. Onthe streets, on the television, on the radio. A hot, angrytangle of shaking fists and ugly words that threatened uslike a monster with a hundred heads. Our country hadchewed us up and spit us out, and now we were beingtreated as if it were our fault.What is one way the author's use of language contributes to the tone of thispassage?O A. The use of figurative language creates a feeling of resentment.O B. The phrase "hot, angry tangle" is used sarcastically to create irony.O c. The word "parade" is used sarcastically to create a lightheartedmood.D. The parallelism draws attention to the confusion of thosereturning. PLEASE HELP! Urgent I'll give brainliest if answer is right tysm. The force,F(Newtons), between two objects is inversely proportional to the square of the distance,d(metres), between them.The force is 0.35 Newtons when the distance between the objects is 8 metres.Work outF(rounded to 2 DP) whend=7metres. cuanta poblacion tiene Janca Solve for z3z-5+2z=25-5z The united nation is compromised of how many countries?1. 1922. 1933. 194 In the diagram to the left ADE is an isosceles triangle. Segment EC bisects DCB.A. Find the measures of DAE, ECB, and EBCB. Explain how you were able to find the measures of these three angles a merchant gains 15% by selling an article for $ 150. by how much does he sell it to double his profit ? what is the ordered pair for the equation -7x+3y=2 Find the solution of y = 6x + 1 for x = 5 A hammer has a mass of 1 kg. What is its weight (i) on Earth (ii) on theMoon (iii) in outer space The radioactive isotope of lead, Pb-209, decays at a rate proportional to the amount present at time t and has a half-life of 3.3 hours. If 1 gram of this isotope is present initially, how long will it take for 70% of the lead to decay? (Round your answer to two decimal places.) Some structures in the eye are listed. 1 cornea 2 iris 3 lens 4 retina Which structures contain light receptors? the information for the questions come with the image, you only need to do one but do as many as you'd like What did a Union general declare inside that state to get him removed from command, angering abolitionists. Cleiton vai participar de uma maratona e para isso treina diariamente correndo 7 km. No final de trs semana quantos quilmetros ter percorrido? The triangles are similar by1-AA2-ASA3-SSS4-SAS i will mark brainliest please help me on this question! Who are the wildfire stakeholders?